2017 AMC 10 B

Complete problem set with solutions and individual problem pages

Problem 1 Easy

Mary thought of a positive two-digit number. She multiplied it by 3 and added 11. Then she switched the digits of the result, obtaining a number between 71 and 75, inclusive. What was Mary's number? (2017 AMC 10B Problem, Question#1)

  • A.

    11

  • B.

    12

  • C.

    13

  • D.

    14

  • E.

    15

Answer:B

Let her 2-digit number be x. Multiplying by 3 makes it a multiple of 3, meaning that the sum of its digits is divisible by 3. Adding on 11 increases the sum of the digits by 1+1=2, and reversing the digits keeps the sum of the digits the same; this means that the resulting number must be 2 more than a multiple of 3. There are two such numbers between 71 and 75:71: and 74 .Now that we have narrowed down the choices, we can simply test the answers to see which one will provide a two-digit number when the steps are reversed:For 71,we reverse the digits, resulting in 17 Subtracting 11, we get 6.We can already see that dividing this by 3 will not be a two-digit number, so 71 does not meet our requirements.Therefore, the answer must be the reversed steps applied to 74.We have the following: 74\to 47\to 36\to 12Therefore, our answer is 12.

Working backwards, we reverse the digits of each number from 71\sim 75 and subtract 11 from each, so we have 6,16,26,36,46The only numbers from this list that are divisible by 3 are 6 and 36. We divide both by 3, yielding 2 and 12. Since 2 is not a two-digit number, the answer is 12.

You can just plug in the numbers to see which one works. When you get to 12, you multiply by 3 and add 11 to get 47. When you reverse the digits of 47, you get 47, which is within the given range. Thus, the answer is 12.

Link Problem
Problem 2 Easy

Sofia ran 5 laps around the 400-meter track at her school. For each lap, she ran the first 100 meters at an average speed of 4 meters per second and the remaining 300 meters at an average speed of meters per second. How much time did Sofia take running the 5 laps? (2017 AMC 10B Problem, Question#2)

  • A.

    5 minutes and 35 seconds

  • B.

    6 minutes and 40 seconds

  • C.

    7 minutes and 5 seconds

  • D.

    7 minutes and 25 seconds

  • E.

    8 minutes and 10 seconds

Answer:C

If Sofia ran the first 100 meters of each lap at 4 meters per second and the remaining 300 meters of each lap at 5 meters per second, then she took \dfrac{100}{4}+\dfrac{300}{5}=25+60=85 seconds for each lap. Because she ran 5 laps, she took a total of 5\cdot 85=425 seconds, or 7 minutes and 5 seconds.

The answer is \rm C.

Link Problem
Problem 3 Easy

Real numbers x,y,and z satisfy the inequalities 0<x<1,-1<y<0 , and 1<z<2. Which of the following numbers is necessarily positive? (2017 AMC 10B Problem, Question#3)

  • A.

    y+x^2

  • B.

    y+xz

  • C.

    y+y^2

  • D.

    y+2y^2

  • E.

    y+z

Answer:E

Notice that y+z must be positive because \left| z\right|\gt\left| y\right|. Therefore the answer is \rm E.

The other choices:

\rm (A) As x grows closer to 0, x^2 decreases and thus becomes less than y.

\rm (B) x can be as small as possible (x>0), so xz grows close to 0 as x approaches 0.

\rm (C)  For all -1<y<0, y>y^2, and thus it is always negative.

\rm (D) The same logic as above, but when -\frac 12<y<0 this time.

Link Problem
Problem 4 Easy

Suppose that x and y are nonzero real numbers such that \dfrac{3x+y}{x-3y}=-2. What is the value of \dfrac{x+3y}{3x-y}? (2017 AMC 10B Problem, Question#4)

  • A.

    -3

  • B.

    -1

  • C.

    1

  • D.

    2

  • E.

    3

Answer:D

Rearranging, we find 3x+y=-2x+6y, or 5x=5y\Rightarrow x=y. Substituting, we can convert the second equation into \dfrac{x+3x}{3x-x}=\dfrac{4x}{2x}=2.

Substituting each x and y with 1, we see that the given equation holds true, as \dfrac{3\left( 1\right)+1}{1-3\left( 1\right)}=-2. Thus,\dfrac{x+3x}{3x-x}=-2.

Let y=ax. The first equation converts into \dfrac{\left( 3+a\right)x}{\left( 1-3a\right)x}=-2, which simplifies to 3+a=-2(1-3a). After a bit of algebra we found out a=1, which means that x=y. Substituting y=x into the second equation it becomes \frac {4x}{2x}=2.

Link Problem
Problem 5 Easy

Camilla had twice as many blueberry jelly beans as cherry jelly beans. After eating 10 pieces of each kind, she now has three times as many blueberry jelly beans as cherry jelly beans. How many blueberry jelly beans did she originally have? (2017 AMC 10B Problem, Question#5)

  • A.

    10

  • B.

    20

  • C.

    30

  • D.

    40

  • E.

    50

Answer:D

Denote the number of blueberry and cherry jelly beans as b and c respectively. Then b=2c and b-10=3(c-10). Substituting, we have 2c-10=3c-30, so c=20, b=40.

From the problem, we see that 10 less than one of the answer choices must be a multiple of 3 and positive. The only answer choice satisfying this is 40. We can check that 40 blueberry and 20 cherry jelly beans indeed does work.

Link Problem
Problem 6 Easy

What is the largest number of solid 2-in \times 2-in \times 1-in blocks that can fit in a 3-in \times 2-in \times 3-in box? (2017 AMC 10B Problem, Question#6)

  • A.

    3

  • B.

    4

  • C.

    5

  • D.

    6

  • E.

    7

Answer:B

We find that the volume of the larger block is 18, and the volume of the smaller block is 4. Dividing the two, we see that only a maximum of four 2 by 2 by 1 blocks can fit inside the 3 by 3 by 2 block. Drawing it out, we see that such a configuration is indeed possible. Therefore, the answer is 4.

Link Problem
Problem 7 Easy

Samia set off on her bicycle to visit her friend, traveling at an average speed of 17 kilometers per hour.When she had gone half the distance to her friend's house, a tire went flat, and she walked the rest of the way at 5 kilometers per hour. In all it took her 44 minutes to reach her friend's house. In kilometers rounded to the nearest tenth, how far did Samia walk? (2017 AMC 10B Problem, Question#7)

  • A.

    2.0

  • B.

    2.2

  • C.

    2.8

  • D.

    3.4

  • E.

    4.4

Answer:C

Let's call the distance that Samia had to travel in total as 2x, so that we can avoid fractions. We know that the length of the bike ride and how far she walked are equal, so they are both \frac {2x}{2}, or x.She bikes at a rate of 17 kph, so she travels the distance she bikes in \frac 7{17} hours. She walks at a rate of 5 kph, so she travels the distance she walks in \frac x5 hours.The total time is \dfrac{x}{17}+\dfrac{x}{5}=\dfrac{22x}{85}. This is equal to \dfrac{44}{60}=\dfrac{11}{15}of an hour. Solving for x, we have:\dfrac{22x}{85}=\dfrac{11}{15},\dfrac{2x}{85}=\dfrac{1}{15},30x=85,6x=17,x=\dfrac{17}{6},Since x is the distance of how far Samia traveled by both walking and biking, and we want to know how far Samia walked to the nearest tenth, we have that Samia walked about 2.8.

Notice that Samia walks \frac {17}5 times slower than she bikes, and that she walks and bikes the same distance. Thus, the fraction of the total time that Samia will be walking is \dfrac{\dfrac{17}{5}}{\dfrac{17}{5}+\dfrac{5}{5}}=\dfrac{17}{22},Then, multiply this by the time \dfrac{17}{22}\cdot44 =34 minutes ,34 minutes is a little greater than \frac 12 of an hour so Samia traveled \sim\dfrac{1}{2}\cdot5=2.5kilometers,The answer choice a little greater than 2.5 is 2.8. (Note that we could've multiplied \frac {34}{60}=\frac {17}{30} by 5 and gotten the exact answer as well)

Since the distance that both rates travel are equivalent, we can find the harmonic mean of the two rates to find the average speed. Therefore, let a=17 and b=5,\overline{s}=\dfrac{2ab}{a+b}\Rightarrow\overline{s}=\dfrac{2\cdot17\cdot5}{17+5}=\dfrac{85}{11},Now, we can find the overall distance by multiply the average speed by the time(hours), 44minutes.Let s=\dfrac{11}{15},d=\overline{s}\cdot t\Rightarrow d=\dfrac{85}{11}\cdot\dfrac{11}{15}=\dfrac{17}{3},Because the distance Samia walks is half the total distance, the distance she walks is \frac {17}6.With some knowledge of sixths, one finds that Samia walked, in kilometers,2.8\overline{3}\approx2.8\Rightarrow \rm C.

Link Problem
Problem 8 Easy

Points A(11,9) and B(2,-3) are vertices of \triangle ABC with AB=AC. The altitude from A meets the opposite side at D(-1,3). What are the coordinates of point C? (2017 AMC 10B Problem, Question#8)

  • A.

    (-8,9)

  • B.

    (-4,8)

  • C.

    (-4,9)

  • D.

    (-2,3)

  • E.

    (-1,0)

Answer:C

Since AB=AC, then \triangle ABC is isosceles, so BD=CD. Therefore, the coordinates of C are (-1-3,3+6)=(-4,9).

Calculating the equation of the line running between points B and D, y=-2x+1. The only coordinate of C that is also on this line is (-4,9).

Link Problem
Problem 9 Easy

A radio program has a quiz consisting of 3 multiple-choice questions, each with 3 choices. A contestant wins if he or she gets 2 or more of the questions right. The contestant answers randomly to each question. What is the probability of winning? (2017 AMC 10B Problem, Question#9)

  • A.

    \frac 1{27}

  • B.

    \frac 19

  • C.

    \frac 29

  • D.

    \frac7{27}

  • E.

    \frac 12

Answer:D

There are two ways the contestant can win.

Case 1: The contestant guesses all three right. This can only happen \dfrac{1}{3}*\dfrac{1}{3}*\dfrac{1}{3}=\dfrac{1}{27} of the time.

Case 2: The contestant guesses only two right. We pick one of the questions to get wrong,3 , and this can happen \dfrac{1}{3}*\dfrac{1}{3}*\dfrac{2}{3} of the time. Thus, \dfrac{2}{27}*3=\dfrac{6}{27}.So, in total the two cases combined equals \frac 1{27}+\frac 6{27}=\rm (D)\frac 7{27}.

Complementary counting is good for solving the problem and checking work if you solved it using the method above.

There are two ways the contestant can lose.

Case 1: The contestant guesses zero questions correctly.

The probability of guessing incorrectly for each question is \frac 23. Thus, the probability of guessing all questions incorrectly is \dfrac{2}{3}*\dfrac{2}{3}*\dfrac{2}{3}=\dfrac{8}{27}.

Case 2: The contestant guesses one question correctly. There are 3 ways the contestant can guess one question correctly since there are 3 questions. The probability of guessing correctly is \frac 13.so the probability of guessing one correctly and two incorrectly is 3*\dfrac{1}{3}*\dfrac{2}{3}*\dfrac{2}{3}=\dfrac{4}{9}.

The sum of the two cases is \frac 8{27}+\frac 4{9}=\frac {20}{27}. This is the complement of what we want to the answer is 1-\frac {20}{27}=\rm (D)\frac {7}{27}.

Link Problem
Problem 10 Easy

The lines with equations ax-2y=c and 2x+by=-c are perpendicular and intersect at (1,-5). What is c? (2017 AMC 10B Problem, Question#10)

  • A.

    -13

  • B.

    -8

  • C.

    2

  • D.

    8

  • E.

    13

Answer:E

Writing each equation in slope-intercept form, we get y=\dfrac{a}{2}x-\dfrac{1}{2}c and y=-\dfrac{2}{b}x-\dfrac{c}{b}. We observe the slope of each equation is \frac a2 and -\frac 2b, respectively. Because the slope of a line perpendicular to a line with slope m is -\frac 1m, we see that \dfrac{a}{2}=-\dfrac{1}{-\dfrac{2}{b}},because it is given that the two lines are perpendicular. This equation simplifies to a=b. Because (1,-5) is a solution of both equations, we deduce a\times1-2\times(-5)=c and 2\times1+b\times(-5)=-c. Because we know that a=b, the equations reduce to a+10=c and 2-5a=-c. Solving this system of equations, we get c=\rm (E)13.

Link Problem
Problem 11 Easy

At Typico High School, 60\% of the students like dancing, and the rest dislike it. Of those who like dancing, 80\% say that they like it, and the rest say that they dislike it. Of those who dislike dancing, 90\% say that they dislike it, and the rest say that they like it. What fraction of students who say they dislike dancing actually like it? (2017 AMC 10B Problem, Question#11)

  • A.

    10\%

  • B.

    12\%

  • C.

    20\%

  • D.

    25\%

  • E.

    33\dfrac{1}{3}\%

Answer:D

60\%\cdot20\%=12\% of the people that claim that they dislike dancing actually like it,and 40\%\cdot90\%=36\%. Therefore, the answer is \dfrac{12\%}{12\%+36\%}=\rm (D)25\%.

Link Problem
Problem 12 Easy

Elmer's new car gives 50\% better fuel efficiency. However, the new car uses diesel fuel, which is 20\% more expensive per liter than the gasoline the old car used. By what percent will Elmer save money if he uses his new car instead of his old car for a long trip? (2017 AMC 10B Problem, Question#12)

  • A.

    20\%

  • B.

    26\frac 23\%

  • C.

    27\frac 79\%

  • D.

    33\frac 13\%

  • E.

    66\frac 23\%

Answer:A

Suppose that his old car runs at x km per liter. Then his new car runs at \frac 32x km per liter, or x km per \frac 23 of a liter. Let the cost of the old car's fuel be c, so the trip in the old car takes xc dollars, while the trip in the new car takes \dfrac{2}{3}\cdot\dfrac{6}{5}xc=\dfrac{4}{5}xc. He saves \dfrac{\dfrac{1}{5}xc}{xc}=\rm (A)20\%.

Because they do not give you a given amount of distance, we'll just make that distance 3x miles.Then, we find that the new car will use 2*1.2=2.4x. The old car will use 3x. Thus the answer is \dfrac{3-2.4}{3}=\dfrac{0.6}{3}=\dfrac{20}{100}=\rm (A)20\%.

You can find that the ratio of fuel used by the old car and the new car in a same amount of distance is 3:2, and the ratio between the fuel price of these two cars is 5:6. Therefore, by multiplying these two ratios, we get that the costs of using these two cars is 15:12=5:4,So the percentage of money saved is 1-\frac 54=\rm (A)20\%.

Link Problem
Problem 13 Medium

There are 20 students participating in an after-school program offering classes in yoga, bridge, and painting. Each student must take at least one of these three classes, but may take two or all three.There are 10 students taking yoga, 13 taking bridge, and 9 taking painting. There are 9 students taking at least two classes. How many students are taking all three classes? (2017 AMC 10B Problem, Question#13)

  • A.

    1

  • B.

    2

  • C.

    3

  • D.

    4

  • E.

    5

Answer:C

By PIE (Property of Inclusion/Exclusion),the answer is 10+13+9-9-20=3.

Link Problem
Problem 14 Medium

An integer N is selected at random in the range 1\leqslant N\leqslant2020. What is the probability that the remainder when N^{16} is divided by 5 is 1? (2017 AMC 10B Problem, Question#14)

  • A.

    \frac 15

  • B.

    \frac 25

  • C.

    \frac 35

  • D.

    \frac 45

  • E.

    1

Answer:D

By Fermat's Little Theorem, N^{16}=\left( N^{4}\right)^{4}\equiv1(mod 5), when N is relatively prime to 5. Hence, this happens with probability \frac 45.

Note that the patterns for the units digits repeat, so in a sense we only need to find the patterns for the digits 0-9. The pattern for 0 is 0, no matter what power, so 0 doesn't work. Likewise, the pattern for 5 is always 5. Doing the same for the rest of the digits, we find that the units digits of 1^{16},2^{16},3^{16},4^{16},6^{16},7^{16},8^{16} and 9^{16} all have the remainder of 1 when divided by 5, so \frac 45.

We can use modular arithmetic for each residue of n (mod 5).

If n\equiv0 (mod 5), then n^{16}\equiv0^{16}\equiv0(mod 5),

If n\equiv1 (mod 5), then n^{16}\equiv1^{16}\equiv0(mod 5),

If n\equiv2 (mod 5), then n^{16}\equiv\left( n^{2}\right)^{8}\equiv\left( 2^{2}\right)^{8}\equiv4^{8}\equiv\left( -1\right)^{8}\equiv1 (mod 5).

If n\equiv3 (mod 5), then n^{16}\equiv\left( n^{2}\right)^{8}\equiv\left( 3^{2}\right)^{8}\equiv9^{8}\equiv\left( -1\right)^{8}\equiv1 (mod 5).

If n\equiv4 (mod 5), then n^{16}\equiv4^{16}\equiv\left( -1\right)^{16}=1 (mod 5).

In 4 out of the 5 cases, the result was 1(mod 5), and since each case occurs equally as 2020\equiv0 (mod 5), the answer is \frac 45.

Link Problem
Problem 15 Medium

Rectangle ABCD has AB=3 and BC=4. Point E is the foot of the perpendicular from B to diagonal \overline{AC}. What is the area of \triangle AED? (2017 AMC 10B Problem, Question#15)

 

  • A.

    1

  • B.

    \frac {42}{25}

  • C.

    \frac {28}{15}

  • D.

    2

  • E.

    \frac {54}{25}

Answer:E

First, note that AC=5 because ABC is a right triangle. In addition, we have AB\cdot BC=2\left[ ABC\right]=AC\cdot BE, so BE=\frac {12}5. Using similar triangles within ABC, we get that AE=\frac 95 and CE=\frac {16}5. Let F be the foot of the perpendicular from E to AB. Since EF and BC are parallel,\triangle AFE is similar to \triangle ABC. Therefore, we have \dfrac{AF}{AB}=\dfrac{AE}{AC}=\dfrac{9}{25}. Since AB=3,AF=\frac {27}{25} . Note that AF is an altitude of \triangle AED from AD, which has length 4. Therefore, the area of \triangle AED is \dfrac{1}{2}\cdot\dfrac{27}{25}\cdot4=\dfrac{54}{25}.

Alternatively, we can use coordinates. Denote D as the origin. We find the equation for AC as y=-\frac 43x+4, and BE as y=\frac 34x+\frac 74. Solving for x yields \frac {27}{25}. Our final answer then becomes \frac 12\cdot \frac {27}{25}\cdot 4=\frac {54}{25}.

We note that the area of ABE must equal area of AED because they share the base and the height of both is the altitude of congruent triangles. Therefore, we find the area of ABE to be \dfrac{1}{2}*\dfrac{9}{5}*\dfrac{12}{5}=\dfrac{54}{25}.

We know all right triangles are 5-4-3, so the areas are proportional to the square of like sides. Area of ABE is \left( \dfrac{3}{5}\right)^{2} of ABC=\frac {54}{25}. Using similar logic in Solution 3, Area of AED is the same as ABE.

Link Problem
Problem 16 Medium

How many of the base-ten numerals for the positive integers less than or equal to 2017 contain the digit 0? (2017 AMC 10B Problem, Question#16)

  • A.

    469

  • B.

    471

  • C.

    475

  • D.

    478

  • E.

    481

Answer:A

We can use complementary counting. There are 2017 positive integers in total to consider, and there are 9 one-digit integers, 9\cdot 9=81 two digit integers without a zero, 9\cdot 9\cdot 9 three digit integers without a zero, and 9\cdot 9\cdot 9=729 four-digit integers starting with a 1without a zero. Therefore, the answer is 2017-9-81-729-729=469.

Link Problem
Problem 17 Medium

Call a positive integer monotonous if it is a one-digit number or its digits, when read from left to right, form either a strictly increasing or a strictly decreasing sequence. For example, 3,23578, and 987620 are monotonous, but 88,7434, and 23557 are not. How many monotonous positive integers are there? (2017 AMC 10B Problem, Question#17)

  • A.

    1024

  • B.

    1524

  • C.

    1533

  • D.

    1536

  • E.

    2048

Answer:B

Case 1: monotonous numbers with digits in ascending order.

There are \sum \nolimits_{n=1}^{9}\left({}\begin{array}{l} {9}\\{n} \end{array}\right) ways to choose n digits from the digits 1 to 9. For each of these ways, we can generate exactly one monotonous number by ordering the chosen digits in ascending order. Note that 0 is not included since it will always be a leading digit and that is not allowed. Also, \varnothing(the empty set) isn't included because it doesn't generate a number. The sum is equivalent to \sum \nolimits_{n=1}^{9}\left({}\begin{array}{l} {9}\\{n} \end{array}\right)-\left({}\begin{array}{l} {9}\\{0} \end{array}\right)=2^{9}-1=511.

Case 2: monotonous numbers with digits in descending order.

There are \sum \nolimits_{n=0}^{10}\left({}\begin{array}{l} {10}\\{n} \end{array}\right) ways to choose n digits from the digits 0 to 9. For each of these ways, we can generate exactly one monotonous number by ordering the chosen digits in descending order. Note that 0 is included since we are allowed to end numbers with zeros. However,\varnothing (the empty set) still isn't included because it doesn't generate a number. The sum is equivalent to \sum \nolimits_{n=0}^{10}\left({}\begin{array}{l} {10}\\{n} \end{array}\right)-\left({}\begin{array}{l} {10}\\{0} \end{array}\right)=2^{10}-1=1023.

We discard the number 0 since it is not positive. Thus there are 1022 here.

Since the 1-digit numbers 1 to 9 satisfy both case 1 and case 2, we have overcounted by 9. Thus there are 511+1022-9=1524 monotonous numbers.

Like Solution 1, divide the problem into an increasing and decreasing case:

Case 1: Monotonous numbers with digits in ascending order.

Arrange the digits 1 through 9 in increasing order, and exclude 0 because a positive integer cannot begin with 0.

To get a monotonous number, we can either include or exclude each of the remaining 9 digits, and there are 2^9=512 ways to do this. However, we cannot exclude every digit at once, so we subtract 1 to get 512-1=511 monotonous numbers for this case.

Case 2: Monotonous numbers with digits in descending order.

This time, we arrange all 10 digits in decreasing order and repeat the process to find 2^{10}=1024 ways to include or exclude each digit. We cannot exclude every digit at once, and we cannot include only 0, so we subtract 2 to get 1024-2=1022 monotonous numbers for this case. At this point, we have counted all of the single-digit monotonous numbers twice, so we must subtract 9 from our total.

Thus our final answer is 511+1022-9=1524.

Link Problem
Problem 18 Medium

In the figure below, 3 of the 6 disks are to be painted blue, 2 are to be painted red, and 1 is to be painted green. Two paintings that can be obtained from one another by a rotation or a reflection of the entire figure are considered the same. How many different paintings are possible? (2017 AMC 10B Problem, Question#18)

  • A.

    6

  • B.

    8

  • C.

    9

  • D.

    12

  • E.

    15

Answer:D

First we figure out the number of ways to put the blue 3 disks. Denote the spots to put the disks as 1-6 from left to right, top to bottom. The cases to put the blue disks are (1,2,3,(1,2,4),(1,2,5),(1,2,6),(2,3,5),(1,4,6). For each of those cases we can easily figure out the number of ways for each case, so the total amount is 2+2+3+3+1+1=12.

Link Problem
Problem 19 Medium

Let ABC be an equilateral triangle. Extend side \overline{AB} beyond B to a point {{B}^{\prime }} so that B{{B}^{\prime }}=3AB. Similarly, extend side \overline{BC} beyond C to a point {{C}^{\prime }} so that C{{C}^{\prime }}=3BC, and extend side \overline{CA} beyond A to a point {{A}^{\prime }} so that A{{A}^{\prime }}=3CA. What is the ratio of the area of \triangle {{A}^{\prime }}{{B}^{\prime }}{{C}^{\prime }} to the area of \triangle ABC? (2017 AMC 10B Problem, Question#19)

  • A.

    9:1

  • B.

    16:1

  • C.

    25:1

  • D.

    36:1

  • E.

    37:1

Answer:E

Solution 1

Note that by symmetry, \triangle A'B'C' is also equilateral. Therefore, we only need to find one of the sides of A'B'C' to determine the area ratio. WLOG, let AB=BC=CA=1. Therefore,BB'=3  and BC'=4. Also,\angle B'BC'=120^\circ , so by the Law of Cosines,B'C'=\sqrt{37} . Therefore, the answer is \left( \sqrt{37}\right)^{2}:1^{2}=37:1.

Solution 2

As mentioned in the first solution, \triangle A'B'C' is equilateral. WLOG, let AB=2. Let D be on the line passing through AB such that A'D is perpendicular to AB. Note that \triangle A'DA is a 30-60-90 with right angle at D. Since AA'=6,AD=3 and A'D=3\sqrt 3. So we know that .db'=11 Note that \triangle A'DB' is a right triangle with right angle at D. So by the Pythagorean theorem, we find A'B'=\sqrt{\left( 3\sqrt{3}\right)^{2}+11^{2}}=2\sqrt{37}.Therefore, the answer is \left( 2\sqrt{37}\right)^{2}:2^{2}=37:1.

Link Problem
Problem 20 Medium

The number 21!=51,090,942,171,709,440,000, has over 60,000 positive integer divisors. One of them is chosen at random. What is the probability that it is odd? (2017 AMC 10B Problem, Question#20)

  • A.

    \frac{1}{21}

  • B.

    \frac{1}{19}

  • C.

    \frac{1}{18}

  • D.

    \frac{1}{2}

  • E.

    \frac{11}{21}

Answer:B

Solution 1

We note that the only thing that affects the parity of the factor are the powers of 2. There are 10+5+2+1=18 factors of 2 in the number. Thus, there are 18 cases in which a factor of 21! would be even (have a factor of 2 in its prime factorization), and 1 case in which a factor of 21! would be odd. Therefore, the answer is \frac 1{19}.

Solution 2

Consider how to construct any divisor D of 21!. First by Legendre's theorem for the divisors of a factorial , we have that there are a total of 18 factors of 2 in the number. D can take up either 0,1,2,3,\cdots, or all 18 factors of 2, for a total of 19 possible cases. In order for D to be odd, however, it must have 0 factors of 2, meaning that there is a probability of 1 case/19 cases=\frac 1{19}.

Link Problem
Problem 21 Hard

In \triangle ABC, AB=6, AC=8, BC=10, and D is the midpoint of \overline{BC}. What is the sum of the radii of the circles inscribed in \triangle ADB and \triangle ADC? (2017 AMC 10B Problem, Question#21)

  • A.

    \sqrt 5

  • B.

    \frac {11}4

  • C.

    2\sqrt 2

  • D.

    \frac {17}6

  • E.

    3

Answer:D

We note that by the converse of the Pythagorean Theorem, \triangle ABC is a right triangle with a right angle at A. Therefore,AD=BD=CD=5, and [ADB]=[ADC]=12. Since A=rs, the inradius of \triangle ADB is \dfrac{12}{\dfrac{5+5+6}{2}}=\dfrac{3}{2 }, and the inradius of \triangle ADC is \dfrac{12}{\dfrac{5+5+8}{2}}=\dfrac{4}{3 }. Adding the two together, we have \frac {17}6.

We have \triangle ABC a right triangle by dividing each side lengths by 2 to create a well known 3-4-5 triangle. We also can know that the median of a right triangle must be equal to half the hypotenuse. Using this property, we have BD=5,CD=5, and AD=5. Now, we can use the Heron's formula to get the area of \triangle ABD as \sqrt{8\left( 2\right)\left( 3\right)\left( 3\right)}=\sqrt{144}=12. Afterward, we can apply this formula again on \triangle ADC to get the area as \sqrt{9\left( 4\right)\left( 4\right)\left( 1\right)}=\sqrt{144}=12. Notice we want the inradius. We can use another property, which is A=rs. This states that Area = Radius(Semiperimeter). (This can be proved by connecting the center of the inscribed circles to the vertices and we can notice the inradius is just the heights of each of the three triangles divided) Finally, we can derive the radii of each inscribed circle. Plugging the semiperimeter and area into the formula, we have 12=8r and 12=9r for \triangle ABD and \triangle ADC, respectively. Simplifying, we have the radii lengths as r=\frac 32and \frac 43. We want the sum, so we have \frac {17}6, or \rm (D).

Link Problem
Problem 22 Medium

The diameter AB of a circle of radius 2 is extended to a point D outside the circle so that BD=3. Point E is chosen so that ED=5 and line ED is perpendicular to line AD. Segment AE intersects the circle at a point C between A and E. What is the area of \triangle ABC? (2017 AMC 10B Problem, Question#22)

  • A.

    \frac{120}{37}

  • B.

    \frac{140}{39}

  • C.

    \frac{145}{39}

  • D.

    \frac{140}{37}

  • E.

    \frac{120}{31}

Answer:D

Solution 1

Notice that ADE and ABC are right triangles.

Then AE=\sqrt{7^{2}+5^{2}}=\sqrt{74},\sin DAE=\sin ABC=\dfrac{5}{\sqrt{74}}=\sin BAE=\dfrac{BC}{4},so BC=\dfrac{20}{\sqrt{74}}. We also find that AC=\dfrac{28}{\sqrt{74}}, and thus the area of ABC is \dfrac{\dfrac{20}{\sqrt{74}}\cdot\dfrac{28}{\sqrt{74}}}{2}=\dfrac{\dfrac{560}{74}}{2}=\dfrac{140}{37}.

Solution 2

We note that \triangle ACB\sim\triangle ADE by AA similarity. Also, since the area of \triangle ADE=\dfrac{7\cdot5}{2}=\dfrac{35}{2} and AE=\sqrt {74},\dfrac{\left[ ABC\right]}{\left[ ADE\right]}=\dfrac{\left[ ABC\right]}{\dfrac{35}{2}}=\left( \dfrac{4}{\sqrt{74}}\right)^{2} , so the area of \triangle ABC=\frac {140}{37}.

Solution 3

As stated before, note that \triangle ACB,\triangle ADE. By similarity, we note that \dfrac{\overline{AC}}{\overline{BC}} is equivalent to \frac 75. We set \overline{AC} to 7x  and \overline{BC} to 5x. By the Pythagorean Theorem, \left( 7x\right)^{2}+\left( 5x\right)^{2}=4^{2}. Combining, 49x^{2}+25x^{2}=16. We can add and divide to get x^2=\frac 8{37}. We square root and rearrange to get x=\dfrac{2\sqrt{74}}{37}. We know that the legs of the triangle are 7x and 5x. Mulitplying x by 7 and 5 eventually gives us \dfrac{14\sqrt{74}}{37}\dfrac{10\sqrt{74}}{37}. We divide this by 2, since \frac 12bh is the formula for a triangle. This gives us \frac {140}{37}.

Solution 4

Let's call the center of the circle that segment AB is the diameter of,O. Note that \triangle ODE is an isosceles right triangle. Solving for side OE, using the Pythagorean theorem, we find it to be 5\sqrt 2. Calling the point where segment OE intersects circle O, the point I, segment IE would be 5\sqrt 2-2. Also, noting that \triangle ADE is a right triangle, we solve for side AE, using the Pythagorean Theorem, and get \sqrt {74}. Using Power of Point on point E, we can solve for CE. We can subtract CE from AE to find AC and then solve for CB using Pythagorean theorem once more (AE)(CE). = (Diameter of circle O+IE ),\left( IE\right)\rightarrow\sqrt{74}\left( CE\right)=\left( 5\sqrt{2}+2\right)\left( 5\sqrt{2}-2\right)\Rightarrow CE=\dfrac{23\sqrt{74}}{37},AC=AE-CE\rightarrow AC=\sqrt{74}-\dfrac{23\sqrt{74}}{37}\Rightarrow AC=\dfrac{14\sqrt{74}}{37},Now to solve for CB:AB^{2}-AC^{2}=CB^{2}\rightarrow4^{2}+\dfrac{14\sqrt{74}^{2}}{37}=CB^{2}\Rightarrow CB=\dfrac{10\sqrt{74}}{37} ,

Note that \triangle ABC is a right triangle because the hypotenuse is the diameter of the circle. Solving for area using the bases AC and BC, we get the area of triangle ABC to be \frac {140}{37}.

Link Problem
Problem 23 Hard

Let N=123456789101112\ldots 4344 be the 79-digit number that is formed by writing the integers from 1 to 44 in order, one after the other. What is the remainder when N is divided by 45? (2017 AMC 10B Problem, Question#23)

  • A.

    1

  • B.

    4

  • C.

    9

  • D.

    18

  • E.

    44

Answer:C

We only need to find the remainders of N when divided by 5and 9 to determine the answer. By inspection,N=4 (mod5),The remainder when Nis divided

by 9is1+2+3+4+\ldots +1+0+1+1+1+2+\ldots +4+3+4+4,but

sin c{{e}^{10=1(mod9)}}we can also write this

a{{s}^{1+2+3+\cdots +10+11+12+\cdots 43+44=\frac{44\cdot 45}{2}=22\cdot 45}}which has a remairder of 0 rod9. Therefoer,by inspection,the answer is (\text{C})9

Note:the sum of the digits or N is270.

Noting the solution above,we try to find the sum of the digits to figure out its remainder when divided by9. From1thru9the sum is4510 thru19the sum is5520 thru29is65,and30thru39is75Thus the sum of the digits is45+55+65+75+4+5+6+7+8=240+30=270and thus N is divisible by9. Now.refer to the above solution ^{N=4\left( mod5 \right)}an{{d}^{ N=0\left( mod9 \right)}}From this information,we can conclude this information, we can conclude that ^{N\equiv 54 \left( mod 5 \right)}and{{ }^{N\equiv 54\left( mod 9 \right)}}.

Therefore ^{N=54\left( md 45 \right)}an{{d}^{N=9\left( md 45 \right)}} so the remainder is (\text{C})9.

Because a number is equivalent to the sum of its digits modulo9we have that ^{N\equiv 1+2+3+4+5+\cdots +44\equiv \frac{44\times 45}{2}\equiv 0(mod9)}Furthemore,we see that N-9ends in the digt5and thus is divisible by5so N-9is divisible by45meaning the remainder is(\text{C})9.

Link Problem
Problem 24 Hard

The vertices of an equilateral triangle lie on the hyperbola xy=1, and a vertex of this hyperbola is the centroid of the triangle. What is the square of the area of the triangle? (2017 AMC 10B Problem, Question#24)

  • A.

    48

  • B.

    60

  • C.

    108

  • D.

    120

  • E.

    169

Answer:C

WLOG, let the centroid of \triangle ABC be I=\left(-1,-1\right), The centroid of an equiateral triangle is the same as the circumcenter. It follows that the circumcircle must intersect the graph exactly three times.

Therefore, A=\left(1,1\right), so AI=BI=CI=2 \sqrt{2}, so since \triangle AIB is isosceles and \angle AIB=120{}^\circ, then by Law of Cosines, AB=2\sqrt{6}. Alternatively, we can use the fact that the circumradius of an equlateral triangle is equal lto \frac{8}{\sqrt{3}}. Therefore, the area of the triangle is \frac{\left(2\sqrt{6}\right)^{2}\sqrt{3}}{4}=6 \sqrt{3}, so the square of the area of the triangle is (\rm C)108.

WLOG, let the centroid of \triangle ABC be G=\left(-1,-1\right). Then, one of the verties must be the other curve of the hyperbola. WLOG, let A=\left(1,1\right). Then, point \rm B must be the refction of \rm C across the

line y=x, so let B=\left(a,\frac{1}{a}\right), and C=\left(\frac{1}{a},a\right),

where a<-1. Because \rm G is the centroid, the average of the x-coordinates of the vertices of the triangle is -1. So we know that a+\frac1a+1=-3, Multplying by a and solving gives us a=-2- \sqrt{3}.

So B=\left(-2-\sqrt{3},-2+\sqrt{3}\right), and C=\left(-2+\sqrt{3},-2-\sqrt{3}\right). So BC=2\sqrt{6}, and finding the square of the area gives us(\rm C)108.

WLOG, let the centroid of \triangle ABC be G=\left(1,1\right) and let point A be \left(-1,-1\right). It is known that the centroid is equidistant from the three vertices of \triangle ABC. Because we have the coordinates of both A and G, we know that the distance from G to any vertice

of \triangle ABC is \sqrt{(1-(-1))^{2}+(1-(-1))^{2}}=2 \sqrt{2}, Therefore,AG=BG=CG=2\sqrt{2}. It follows that from \triangle ABG, where AG=BG=2 \sqrt{2} and \angle AGB= \frac{360^{\circ}}{3}=120^{\circ}\left[ \triangle ABG \right] = \frac{(2 \sqrt{2})^{2}\cdot \sin\left(120\right)}{2}=4 \cdot \frac{\sqrt{3}}{2}=2 \sqrt{3} using the formula for the area of a triangle with sine  \left(\left[\angle ABC \right] = \frac{1}{2}AB \cdot BC \sin\left(\angle ABC\right)\right).

Because \triangle ACG and \triangle BCG are congruent to \triangle ABGthey also have an area of 2\sqrt{3}.

Therefore \left[\triangle ABC\right]=3\left(2 \sqrt{3}\right)=6 \sqrt{3}.Squaring that give us the answer of \left(\rm C\right)108.

WLOG, let the centroid of the triangle be (1,1). By symmetry, the other vertexis (-1,-1). The distance between these two points is 2\sqrt{2}, so the height of the triangle is 3\sqrt{2}, the side length is 2\sqrt{6}, and the area is 6\sqrt{3}, yielding an answer of (\rm C)108.

Link Problem
Problem 25 Hard

Last year Isabella took 7 math tests and received 7 different scores, each an integer between 91 and 100, inclusive. After each test she noticed that the average of her test scores was an integer. Her score on the seventh test was 95. What was her score on the sixth test? (2017 AMC 10B Problem, Question#25)

  • A.

    92

  • B.

    94

  • C.

    96

  • D.

    98

  • E.

    100

Answer:E

Since all of the scores arefrom 91- 100, we can 'subtract' 90 off from all of the scores. Basically, we're looking at the units digits except for 100; we're looking at 10 in this case. Since the last score Was a 95, the sum of the scores from the first six tests must be 2 mod 7 and 0 mod 6. Trying out a few cases, the only solution possibleis 30 (this is from adding numbers 1-10). The sixth test score must be 0 mod 5 because 30≡0 mod 5. The only possible test scores are 95 and 100, so the answer is \boxed{\left( \text{E}\right)100}.

Link Problem
Table of Contents
  • 1
  • 2
  • 3
  • 4
  • 5
  • 6
  • 7
  • 8
  • 9
  • 10
  • 11
  • 12
  • 13
  • 14
  • 15
  • 16
  • 17
  • 18
  • 19
  • 20
  • 21
  • 22
  • 23
  • 24
  • 25